Anybody know why the correct Answer choice B weakens the argument??I just can't seem to see how communal objects being passed down from one generation to the next weakens the argument.
I did not understand how and why he figured out which parts of the sentences he could use to make conditional statements from the video. He did not use conditional logic for the first sentence even though it had key words "not" and "are".
I had this down to B or D during the timed exam, and I can't figure out what is technically wrong with B. In my mind, it's as close to a sufficient assumption as D is.
https://7sage.com/lsat_explanations/lsat-73-section-2-question-21/
Hi, I was wondering whether anyone could explain why E is correct.
I was not sure about E because it says more than half of the students received a grade of B-or higher, but I ...
Hello,
I have a question about these two choices
1. the argument takes for granted that the higher sales of established products are due to effective advertising
2. the argument confuses a condition necessary for increasing product ...
Hi guys I would appreciate it if someone could review my reasoning for this question and let me know if this is correct. When BRing I realized that I made a mistake and chose A rather than E which is correct.
Could someone be so kind to explain why AC B is correct? I went with E, quite confidently at that. Any insight is greatly appreciated. Thanks in advance!
I know it's a bad idea to argue with LSAT answers, but I find it's the best way to improve - I want to really understand exactly why my thinking is wrong. I can't seem to find out an adiqute explination for why D is wrong for this question, would love ...
I understand the correct answer. However, it has not clicked why option A is wrong. Doesn't the argument presume that the components for the architectural style also have the same qualities. My reason is because, it conclusion is drawn form elements of ...
Aren't there two main ways to weaken an argument? Either by going for the premises (contradicting them) or showing why the conclusion doesn't necessarily follow from them? I thought C did the first, but now I am having doubts. The stimulus concludes that ...
My issue with this stimulus is about the term consent versus consult. If ANY of the members had said "No" to the release of this report, would that still mean the chairperson consulted them?
So I had a really difficult time with this question because of answer choice A. When it says "takes for granted," is that saying the author is assuming the information in this choice to be true? And why isn't the information in A pointing out a flaw? If it ...
Hi,
I can see why (B) is correct but I cannot figure out why (C) is wrong. Referring to Line 32 - 34, I thought jazz purists don't like jazz music be play with electic piano. Thus, (C) will also weaken author's characterization on the purists. ...